ARDMS registry

  1. Describe the normal aortic valve anatomy?
    It is comprised of three cup-shaped leaflets, the right, the left and the non-coronary cusp. Behind each of the cusps the aortic wall dilates to become the Sinus of Valsalva. Behind the right and left cusp are the coronary Ostia for the left and right coronary arteries.
  2. Why is the Aortic valve so resistant to regurgitation?
    compared to the mitral leaflets, the aortic leaflets are unsupported. Never the less they are partly surrounded by the hearts fiberous skeleton, which strengthens the aortic annulus. Because the leaflets are cupped shape they overlap and support each other. The aortic valve is very resistant to regurgitation.
  3. What is the normal area of the aortic valve?
    The normal area ranges from 2.5cm2 to 3.5 cm2.
  4. What is the normal gradient across the aortic valve during systole? what is the normal aortic valve flow velocity?
    The normal gradient is 2-4 mmHg in adults. The normal flow velocity is 1.4m/sec, w/ a range of 0.9 to 1.8 m/sec.
  5. Name the branches of the Aortic starting w/ the one closest to the aortic valve?
    The branches of the aorta are the innominate(Right Brachial-cephalic), the Left Common Carotid, and the Left subclavian arteries.
  6. What are the primary M-mode findings associated w/ aortic stenosis?
    • Multiple reverberant echoes during systole and diastole, owing to the thickening of the leaflets,
    • decreased separation of the valve leaflets,
    • left ventricular hypertrophy.
  7. What are the primary 2-D echo findings of aortic stenosis.
    • 2-D finding of Aortic Stenosis are:
    • thickening of the leaflets, w/ decreased mobility
    • left ventricular hypertrophy
    • occasional post stenotic dilatation of the aorta.
  8. In aortic stenosis, what changes are seen in the spectral Doppler trace?
    I aortic stenosis, the Doppler spectral trace shows increased velocity and turbulence (spectral broadening). In severe stenosis, the time from onset of flow to peak flow velocity is prolonged.
  9. How does peak aortic valve gradient correlate with the severity of stenosis?
    If cardiac output is normal, a peak aortic valve gradient of more than 100mmHg denotes severe stenosis. If cardiac output is low, a valve area may be critically small, but the gradient may be as low as 3mmHg. Thus, the clinician needs to know the valve area as well as the gradient.
  10. What is the best non-invasive method for quantifying aortic valve stenosis?
    • The best noninvasive method to use is the continuity of flow equation to calculate the area of the aortic valve.
    • CSA1*V1=CSA2*V2
  11. How does the aortic valve area correlate to severity of stenosis?
    Mild >1.5cm2, Moderate 1.0 -1.5 cm2, Severe < 1.0 cm2.
  12. How does AS effect the ventricles?
    • As causes pressure overload of the left ventricle.
    • The ventricle responds to this overload (increased wall stress) by becoming hypertrophied. Overtime, the pressure overload will cause left ventricular dilatation and decreased contractility.
    • The R. ventricle is usually not affected.
  13. How does As affect the atria?
    • As Left LV thickens, ventricular compliance decreases and atrial pressures increase, leading to Left atrial enlargement. As LV systolic pressure rises, the end-diastolic pressures also rise.
    • The R. atrium is usually not affected.
  14. How does AS affect the great vessels?
    AS sometimes produces post stenotic dilatation of the aorta because of the high velocity aortic jets impact on the aortic walls. The pulmonary artery is usually not affected.
  15. Which of the following methods is the most accurate means of calculating the aortic valve area?
    M-Mode measurements of the cusp separation.
    2-D planimetry of the Ao area in PSAX.
    Doppler calculation of the continuity of flow equation.
    Doppler calculation of flow equation is the most accurate in determining Ao valve area.
  16. 2-D echo finds a 58 year old man w/ a systolic murmur reveals the following findings:
    concentric LV hypertrophy
    mild-moderate thickening of the AO leaflets w/ decreased opening
    systolic doming of the AO leaflets in the PLAX view.
    What is the most likely diagnosis?
    • The most likely diagnosis is stenosis of the AO due to a bicuspid ao valve.
    • Thickened leaflets and concentric hypertrophy may also be seen in pt. w/ ao stenosis, but the systolic doming in the PLAX view is typical of the bicuspid AO valve.
  17. Define Aortic Sclerosis?

    Define Aortic Stenosis?
    Ao sclerosis denotes a hardening and fibrosis of the AO leaflets. This condition does not produce a significant gradient, it may cause a systolic murmur or some degree of regurge.

    Ao stenosis denotes narrowing of the leaflets or outflow tract. this condition is different from sclerosis in that it stenosis implies a presence of a hemodynamic gradient.
  18. What are the primary 2-D echo findings associated w/ congenital bicuspid Ao valve stenosis?

    What would be the major differences in findings between degenerative AO stenosis and rheumatic Ao valve stenosis?
    • The typical findings associated w/ congenital bicuspid Ao valve stenosis are:
    • concentric LV hypertrophy
    • mild-moderate thickening of the AO leaflets
    • systolic doming of the AO leaflets in PLAX

    Although the AO valve might present similar appearances in both cases, rheumatic heart disease is almost always accompanied by coexisting mitral stenosis.
  19. What is Takayasu's Arteritis?

    In a pt. w/ Takayasu Arteritis what would you look for in an echo?
    • AKA: Aortic Arch syndrome there is fibrosis of the AO arch and the Desc. Ao. this disease occurs more frequently in women of Asian/African descent.
    • On an echo you wopuld look for supravalvular aortic stenosis. In the most severe forms this disease may present w/ multiple coarctations of the Ao arch system.
  20. How does AO regurge affect the stenotic AO valve gradient?
    How does Ao regurge affect the AO valve area by the continuity of flow equation?
    • mild AO regurge has no affect ofn the AO valve gradient. Mod-severe AO regurge causes the existing gradient to increase due to the volume overload of the LV.
    • AO regurge does ot affect the valve area by the continuity equation because V1=V2.
  21. What are the main echo findings associated w/ chronic AO regurge?
    What are the main echo finding assoc. w/ acute AO regurge?
    • Mild chronic Ao regurge may have slight AO valve thickening and otherwise normal echo exam. Mod-severe AO regurge may present w/ early LV dilatation and hypercontractility and in late disease impairment of the Lv function.
    • Mild Acute AO may have a fairly nl. echo.
    • Mod-severe Acute AO will have hypercontractile LV, LV dilatation, and occasional premature closure of the MV due to increased ED pressures. These pt.'s usually have some pathological AO condition such as veggies, prolapse, dissection or trauma.
  22. How is regurge quantified w/ color flow Doppler?
    CF Doppler "maps" the area of regurge flow. The larger the jet, the more severe the regurge, one must also take into account the total size, length, and width of the jet.
  23. How does the Ao regurge affect the atria?

    How does it affect the great vessels?
    • Ao regurge does not affect the R. atrium. when chronic regurge and decreased LV compliance is present it results in LV dilatation, the LA may enlarge slightly.
    • AO regurge does not affect the the great vessels. Although dilatation of the Ao causes AO valve regurge, Ao valve regurge does nat cause dilatation of the Aorta.
  24. How does Ao regurge affect the ventricles?

    What is the most common congenital cause of AO regurge?
    • Mild Ao regurge does not affect the ventricles. Mod-severe regurge results in LV dilatation due to volume overload. In such cases the LV contractility is hyperdynamic. The LV continues to dilate until decompensation sets in and ventricular function decreases.
    • Bicuspid aortic valve is the most common congenital cause of AO regurge.
  25. What is another word for diastolic reverse flow in the descending AO?

    How is diastolic flow reversal used to quantitate Ao regurge?
    Another word for diastolic flow reversal is retrograde flow.

    In mild Ao regurge you will usually not see diastolic flow reversal in the desc. AO. In moderate regurge you will usually see early diastolic flow reversal. In severe Ao regurge you will usually see holodiastolic flow reversal in the Desc. AO.
  26. Name the 3 classic M-mode findings in AO regurge?
    • The 3 classic M-mode findings are diasti=olic fluttering of the AO leaflets
    • diastolic fluttering of the intraventricular septum
    • dilatation & hypercontractility of the LV( hypcontractility may be present if LV decompensation is present)
    • mitral valve preclosure (in severe acute AO regurge).
  27. In Ao endocarditis what symptoms are pts likely to present with?
    What is the common cause of flail Ao leaflet?
    • Symptoms of AO endocarditis include:
    • fever
    • chills/night sweats
    • a diastolic murmur
    • tachycardia
    • dyspnea on exertion or @ rest

    Most common cause of flail AO leaflet is endocarditis. A less common cause is trauma.
  28. Define Marfan's syndrome?

    How does Marfan's affect the Ao valve, how does it affect the mitral valve?
    • Marfan's is a connective tissue disorder, characterized by inc. joint flexibility  and elongation of the bones. Ocular lens problems and cardiac abnl. are often present.
    • Cardiac manifestations of Marfan's syndrome include Asc. Ao dilatation 7 mitral valve prolapse. Depending on the severity of the disease varying degrees of AO/Mv regurge may be present. AO dissection may also occur.
  29. Describe the anatomy of the Tricuspid valve, including name and location of each leaflet?
    • The tricuspid valve is located between the R. atrium and R. ventricle. It has 3 leaflets: the anterior, posterior, and medial(septal) leaflets.
    • The medial(septal) leaflet is attached to the septum. It's location is closer to the cardiac apex than that of the anterior mitral leaflet.
  30. Name 3 classic M-mode findings associated w/ tricuspid stenosis?
    Describe the most common 2-D echo findings associated/ tricuspid stenosis?
    • 3 classic M-mode findings associated w/ Tricuspid stenosis are:
    • decreased E to F slope
    • a reduced early diastolic amplitude
    • multiple reverberant echos during systole and diastole
    • The most common 2-D echo findings are:
    • thickening and tethering (doming) of the tricuspid leaflets
    • decreased leaflet mobility during diastole
    • mitral stenosis
  31. What is the nl. flow velocity (mean value and range) through the tricuspid valve?
    Describe the changes in the Doppler spectral trace associated w/ tricuspid stenosis?
    • The nl. flow velocity through the tricuspid valve is a mean of 0.6m/sec, w/ a range of 0.4-0.8 m/sec.
    • In tricuspid stenosis, changes in the spectral Doppler trace include:
    • an increase in flow velocity
    • an increase in flow turbulence(detected w/ PW or CW Doppler imaging)
    • a decrease in the rate of drop-off for early diastolic flow (pressure 1/2 time).
  32. How does tricuspid stenosis affect the atria?

    How does tricuspid stenosis affect the ventricles?
    • Mild tricuspid stenosis usually does not affect the atria. Mod-severe tricuspid stenosis causes R. atrial enlargement. Increased atrial pressure can also produce peripheral edema. The LA is not usually affected by tricuspid stenosis.
    • In severe tricuspid stenosis the RV may appear smaller than normal because CO is reduced. The LV is rarely affected by tricuspid stenosis.
  33. What are the main echo findings of chronic tricuspid regurge??
    Mild TR may yield nl. echo findings, Mod-severe TR may cause RV volume overload. The RV becomes dilated, flattening of the IVS may be evident. Due to increased volume the RA is dilated and the vena cava is distended.
  34. What are the main echo findings of acute TR?
    Mild acute TR may yield normal findings, more likely the evidence will show some evidence of valvular disease (such as trauma related prolapse, valvular disruptions, or vegetative lesions). Mod-severe regurge may cause hyperdynamic motion and mild dilatation of the RV. The RV does not have time to enlarge, as it does in chronic regurge. The RA is slightly dilated and the IVC is distended.
  35. During pulsed Doppler exam, how does tricuspid regurge appear on the spectral trace?
    How is TR quantified w/ a pulsed Doppler instrument?
    • TR appears as turbulent (broad spectrum) systolic flow w/in the RA. Because the velocity of the TR jet is not as high as that of MR, the spectral trace may not alias.
    • TR is quantified by mapping the area of systolic turbulence in the RA. The larger the area of the jet the more severe the regurge.
  36. How is TR quantified by color flow Doppler?
    CF Doppler imaging maps the area of reguge flow. The larger the jet, the more severe the regurge, One must also note the total size, length, and width of the jet.
  37. If you already know the peak tricuspid regurge velocity, how can you calculate the RVSP?

    What is the significance of this calculations?
    To calculate the RVSP, add the TR velocity (once converted 4v2) w/ the estimated RA pressure. 4v2+RAP=RVSP

    This calculation is a noninvasive means of calculating the PA pressures. In the absence of PS, PA pressures will be the same as the RVSP.
  38. What is carcinoid syndrome?

    How does carcinoid syndrome cause TR?
    • Carcinoid syndrome is associated w/ carcinoid tumors of the intestinal tract or pancreas. Such tumors release serotonin, that is deposited on the endocardial walls and valvular surfaces.
    • Seretonin deposits build on the tricuspid or pulmonic leaflets, causing them to become thickened and immobile, so that they cannot coapt properly. This often results in moderate to severe regurge. In some pts. the valves become immobile in the closed position creating stenosis. The left side of the heart is rarely affected due to the lungs filtering out the serotonin.
  39. Define Tricuspid valve prolapse?

    What is the relationship between TV prolapse and MV prolapse?
    TV prolapse denotes systolic displacement of one or more of the tv leaflets into the R atrium. The anterior & septal leaflets prolapse more frequently than the posterior leaflet.

    Almost 90% of pts. w/ TV prolapse also have MV prolapse.
  40. What is the incidence of TV stenosis in rheumatic heart disease?
    Approx. 10% of pts. w/ rheumatic heart disease have some degree of TV stenosis. The symptoms  of TV stenosis may be masked by mitral stenosis, which is almost always present.
  41. Desc. the anatomy of the pulmonic valve, including the name and location of each leaflet?
    The pulmonic valve is the most anteriorly placed cardiac valve. It lies w/in the RVOT, to the left of the aortic valve. The pulmonic valve has three leaflets: the anterior, the right(posterior), and the left. The R. leaflet is also referred to in m-mode books as the posterior leaflet.
  42. What is the normal flow velocity through the pulmonic valve, as assessed by Doppler US in both children and adults?
    The normal pulmonic flow gradient is 1-3 mmHg. In adults, the normal pulmonic flow velocity is amean of 0.7 m/sec, w/ a range of 0.5-0.9 m/sec. In children, the normal mean pulmonic flow velocity is slightly higher, at 0.8 m/sec.
  43. What is the primary m-mode finding associated w/ pulmonic stenosis?

    What are the primary 2-D echo findings associated w/ pulmonic stenosis?
    The primary M-mode findings assoc. w/ PS is an increase of more than 7 mm in the "a" dip.

    • The primary 2-D echo findings in PS are:
    • valve thickening
    • decreased leaflet excursion
    • systolic doming
    • RV hypertrophy
    • poststenotic dilatation of the pulmonary artery
  44. During the Doppler examination, what changes does PS produce in the Doppler spectral trace?

    How does the peak pulmonic gradient correlate w/ the severity of PS?
    In PS the Doppler spectral trace shows increased flow velocity and turbulence (spectral broadening). In severe stenosis, the interval from the onset of flow to peak velocity is prolonged as well as a high velocity jet.

    If the cardiac output is normal, a gradient of more than 75 mmHg denotes severe stenosis. If CO is low, the valve area may be critically small, but the gradient may be as small as 3 m/sec (36mmHg).
  45. How does PS affect the ventricles?
    PS causes pressure overload of the RV. In response to this overload (increased wall stress), the ventricle becomes hypertrophied. Overtime the pressure overload results in dilatation of the ventricle along w/ decreased contractility. The LV is usually not affected.
  46. How does PS affect the atria?
    Thickening of the RV causes the RV compliance to decrease and the RA pressure to increase, leading to RA enlargement. As the RV systolic pressure rises, the end-diastolic pressure also rises. The LA is not often affected.
  47. How does PS affect the great vessels?
    PS may result in poststenotic dilitation of the pulmonary artery, due to the high velocity jet. The aorta is rarely affected.
  48. What is the most common cause of PS?

    How does carcinoid syndrome cause PS? How does it cause PI?
    The most common cause of PS is a congenital abnormality.

    Serotonin builds up on the pulmonic  and/or the tricuspid leaflets, causing them to become thickened and immobile. If they become fixed in the closed position, stenosis will result. If the become fixed in the open position (as is usually the case) regurge will ensue.
  49. What are the main echo findings associated with chronic PI?
    In trivial or mild PI, the echo findings may be normal. Mod-severe PI is indicated early by RV dilatation, paradoxical septal motion & hypercontractility. Later in the disease process impairment of RV function appears.
  50. What is the most common cause of PI?
    PI usually affects valves that are otherwise normal. About 60% of the general population has some degree of PI, probably owing to altered pulmonary artery geometry. Because the artery lies across the aorta it changes shape from circular to oblong at this site. Furthermore, because pulmonic pressure is normally low the valve may not close tightly.
  51. How does PHTN affect pulmonic regurge?
    PHTN causes the peak regurge velocity to increase. Normally, the regurge velocity is about 1 m/sec. In PHTN, the regurge jet velocity may be greater than 4 m/sec.
  52. How does PI affect the ventricles?
    Mild PI does not affect the RV. Mod-sever PI results in RV volume overload resulting in dilatation and hyperdynamic contractility. The RV eventually becomes decompensated and ventricular function may decrease. The LV is not affected.
  53. How does the PI affect the atria?

    How does it affect the great vessels?
    In chronic PI the RA may enlarge slightly because of decreased RV compliance. PI does not affect the LA.

    PI does not affect the great vessels.
  54. Describe the classic M-mode findings  assoc. w/ mitral valve endocarditis?
    • Classic M-mode findings of MV endocarditis are:
    • nl. unrestricted valve motion
    • an echogenic shaggy mass on the anterior or posterior leaflet, if the vegetation is large and mole, this mass may move independently of the leaflet.
  55. What are the classic m-mode findings of Aortic valve endocarditis?
    • The normal m-mode echo findings of aortic valve endocarditis are:
    • nl. unrestricted valve motion
    • an echogenic shaggy mass that appears during diastole and disappears during systole ( if the vegetation is large and mobile, it may be seen during diastole above the AMVL, in the LVOT).
  56. Describe the typical 2-D echo appearance and location of the MV vegetations in endocarditis?
    • In MV endocarditis, a typical 2-D study may show:
    • thick redundant leaflets
    • mass lesions on the flow (atrial) side of the leaflets
    • mobile mass in the LA during systole and in in the LV in diastole.
  57. Describe the typical 2-D echo appearance and location of the aortic valve vegetations in endocarditis?
    • In aortic valve endocarditis a 2-D echo may show:
    • thick, redundant leaflets
    • mass lesions on the flow  (ventricular) side of the leaflets
    • mobile mass in the LVOT during diastole and in the aorta during systole.
  58. Why are intravenous drug abusers at risk for endocarditis involving the R side of the heart?

    What is the most common organism seen in intravenous drug abusers?
    • Intravenous drug abusers are at risk for endocarditis involving the R side of the heart because they use contaminated needles or syringes to make multiple injections into the venous system.
    • Staphylococcus aureus is the most common organism seen in the intravenous drug abusers.
  59. Define acute bacterial endocarditis.

    Define subacute bacterial endocarditis.
    • Acute bacterial endocarditis denotes infection of a normal valve. Before antibiotics became widely available, patients w/ acute bacterial endocarditis usually died w/in 6 weeks.
    • Subacute bacterial endocarditis denotes infection of an abnormal valve. Patients w/ mitral valve prolapse, prosthetic cardiac valves, or rheumatic heart disease are at risk for this type of endocarditis.
  60. On 2-D echos what are the main differences between mechanical and bioprosthetic  valves?
    • W/ mechanical prosthetic valves, more valve masking is present, and more reverberations emanate from the valve disc, leaflet or ball.
    • W/ bioprosthetic valves, some masking is present; because the central area of these valves are fabricated of a biologic material, however, comparatively few reverberations emenate from this area.
  61. From the patients standpoint, what are the main differences between mechanical and bioprosthetic valves?
    Mechanical valves are extremely durable (some upto 20 years w/o complications) but necessitate lifelong anticoagulation therapy. Although bioprosthetic valves are less durable (10-12years) they do not require anticoagulation. Mechanical valves also make more noise (especially the caged ball valve) than bioprosthetic ones.
  62. Name a commonly used caged-ball prosthetic valve?
    What are the main advantages and disadvantages of the caged-ball valve?
    • The Starr-Edwards valve is a commonly used caged-ball prosthetic valve.
    • The advantages of the caged ball valve are their durability. Starr-Edwards commonly last for 15-20 years. The disadvantage is that pts. must take lifelong anticoagulants. Caged-ball valves also have a higher transvalvular  gradient than other prosthetics.
  63. Name a caged-disc prosthetic valve?

    What are the advantages and disadvantages of a caged-disc valve.
    • The Beall valve is a caged-disc prosthesis. Although such valves have a lower profile (height) than the Starr-Edwards caged-ball valve, they are no longer used.
    • The main advantage of the caged-disc valves is their durability and low profile. For example, in the mitral position, the cage of the caged-disc valve does not protrude into the LVOT as far as the Starr-Edwards valve. The disadvantages of the caged-disc valves include a high transvalvular gradient and the fact that they cause more hemolysis than other prosthetic valves.
  64. Name a commonly used tilting disc prosthetic valve?
    Name the advantages and disadvantages of the tilting-disc valves?
    • The Bjork-Shiley valve is a commonly used tilting-disc prosthesis. such valves have a much better flow dynamics than valves that have a central occlude such as the Starr-Edwards caged ball prosthesis.
    • The advantage of the tilting-disc prosthesis is that they offer better flow dynamics by a low transvalvular gradient and decreased turbulence. The disadvantages are questionable durability and more prosthetic regurgitation than is associated with other valves.
  65. What is the one thing that all mechanical prosthetic valves share with each other?
    All mechanical prosthetic valves cause some degree of stenosis and regurgitation
  66. How is a porcine prosthetic valve made?
    Porcine valves are preserved in glutaraldehyde and then attached to a polypropylene stent, which had a Dacron sewing ring. Because the preserved leaflets are nonviable, valve rejection is not a problem.
  67. What are the advantages and disadvantages of a porcine prosthetic valve.
    The main advantage is their low thrombogenicity. Therefore unlike mechanical valves porcine bioprosthesis valves do not need anticoagulation therapy.  All but the smaller vavle sizes have low transvalvular gradients. On the other hand porcine valves offer less durability and longevity than mechanical valves. Over a 5-10 year period the porcine leaflets become thickened and stenosed.
  68. Describe the normal appearance of a porcine valve as observed w/ 2-D echo imaging?
    In the PLAX view the porcine valve shows two of the valve stents. Because the valvular reverberations are minimized, it is often possible to see leaflet movement between the stents. The apical views usually offer  better picture of the leaflet morphology and motion, because in these views, the US beam is perpendicular to the valve leaflets.
  69. Describe the appearance of flow through a nl. porcine mitral valve, as observed w/ color flow Doppler?
    The AP views are best for assessing the mitral flow w/ color flow Doppler technique, because forward flow is directed towards the LV apex. As it passes through the valve, the red color jet is centrally located and mostly laminar in appearance.
  70. In using M-mode echos to evaluate the fxn. of the caged ball (Starr-Edwards) prosthetic mitral valve, where should you place the transducer?

    If the caged-ball valve were in the AO position, where would you place the transducer?
    • In evaluating the caged-ball (Starr-Edwards) prosthetic mitral valve, the M-mode tx should be placed at the LV apex, so the full excursion of the leaflets can be documents as the ball moves into the open position. This view also allows evaluation of the ball's timing.
    • In assessing the caged-ball (Starr-Edwards) prosthetic AO valve, the M-mode tx should be placed above the valve, either in the suprasternal or the right supraclavicular window. This placement will allow the ball's excursion & timing to be documented.
  71. Describe the 2-D echo appearance of the caged-ball ( Starr-Edwards) mitral valve, as seen in the PLAX view?
    In the PLAX view the valve's cage which is a fixed position w/in the LV cavity. The LV side of the ball is easily visualized as it moves w/in the cage. The atrial (far) side of the ball appears w/in the LA. This artifact is caused by a delay that occurs when the US beam crosses the gas-filled ball. The slowing of th US beam causes the "other side," of the ball to appear incorrectly positioned in the LA.
  72. Describe the appearance of flow through a nl. mitral caged-ball valves, as observed w/ color flow Doppler?
    Color flow Doppler assessment of the mitral caged-ball valve normally shows flow on both sides of the ball. If flow is detected only on one side, a thrombus or vegetative mass should be detected.
  73. What is the major limitation of the chest wall echo in evaluating a prosthetic mitral valve?
    In evaluating a prosthetic MV  the major limitation of the chest wall evaluation is valve "masking", which makes it difficult to detect regurgitation. Even regurgitation is seen, "masking," (shadowing) prohibits accuarate quantitation. TEE echos allow excellent evaluation of the atrial side of the prosthetic MV and is the best technique for evaluating regurgitation.
  74. Name the five complications of the prosthetic valve dysfunction?
    • The 5 main prosthetic valve dysfunctions w/ prosthetic valves are:
    • perivalvular leakage
    • bioprosthesic stenosis/regurgition
    • valve dehiscence/strut failure
    • ring abscess
    • thrombus formation
    • endocarditis
    • hemolysis
  75. How would you evaluate a prosthetic valve for possible stenosis?
    Evaluation of prosthetic valve stenosis is difficult. Because prosthetic valves have a wide range of nl. gradients, depending on valve type, valve size, and CO. Ideally a previous Doppler study will be available for comparison of cardiac function, heart rate & valve gradient. Nl. transvalular gradient in prosthetic valves ranges from 3-7 mmHg, and is 14-20 mmHg for prosthetic Ao valves.
  76. Name and describe the 2 main layers of the pericardium?
    The pericardium consists of a visceral and a parietal  layer. The visceral layer lies directly upon the external surface of the heart and is commonly referred to as the epicardium. The parietal or fibrous pericardium is the thick outer sack. The pericardial cavity lies between the two layers.
  77. What are the three layers of the pericardium?
    Most books refer to only 2 layers of the pericardium, there are three anatomical layers to the pericardium. The serous visceral (covering the outer surface of the heart-epicardium), the serous parietal which lines the inside of the fibrous pericardium (the parietal or thick outer sack).
  78. In 2-D echoes, what is the best way of differentiating between pericardial effusions and pleural effusions?

    What is the function of the pericardium?
    • The desc. aorta is the landmark, pericardial effusions will be seen between the LA and the desc. ao; in some pts. the desc. ao will be displaced posteriorly. Pleural effusions will be inferior and posterior to the desc. ao and will not displace the desc. ao away from the LA.
    • In general, the pericardium limits ventricular filling, reduces friction that results from cardiac motion, and may act as a barrier to infectious organisms.
  79. Define pericarditis?

    What are the three classical physical findings associated w/ pericarditis and or pericardial effusions?
    • Pericarditis is an inflammation of the pericardium. In response to this inflammation , the visceral pericardium exudes serous fluid. Pericarditis is more common in men than women, it is more prevalent in adults than in young children.
    • The 3 classical findings associated w/ pericarditis +/or pericardial effusion are:
    • chest pain
    • pericardial friction rub
    • dyspnea
  80. Name 8 etiologies of pericardial effusions?
    • Pericardial effusions may be caused by:
    • an idiopathic, or nonspecific, disorder (the most common cause)
    • viral infection
    • bacterial infection
    • uremia
    • radiation therapy
    • acute myocardial infarction
    • Dressler's syndrome, or delayed postmyocardial infarction
    • postpericardiotomy syndrome
  81. What are two etiologies of loculated pericardial effusions?
    Loculated pericardial effusions are quite rare but may occur in pts. who have metastatic disease with pericardial involvement. Pericardial infiltration is most often seen in lung or breast malignancy. as tumor cells invade the pericardium, sections of the pericardium become walled off, and fluid can become loculated. Sometimes, loculated effusions also occur after cardiac surgery. In cases, pericardial adhesion may result in loculated accumulations of fluid.
  82. Define cardiac tamponade?
    Cardiac tamponade is an impairment of diastolic fillin, caused by an increase in intracardial pressure. Tamponade most often, results from moderate-large pericardial effusion, although it may result from a small, rapidly accumulated effusion (as when a vebtricle is accidently perorated during cardiac catherterization.)  Small acute  cases of cardiac tamponade may result in more serious cardiac conditions than larger chronic cases of tamponade.
  83. What are 3 classic physical findings associated w/ cardiac tamponade?
    • 3 classic physical findings of cardiac tamponade are:
    • pulsus paradoxus, which causes a > 10-mmHg decrease in systolic blood pressure during inspiration.
    • tachycrdia
    • dyspnea
    • Beck's triad (elevated venous pressure, hypotension, and a quiet precordium)
  84. What are the 2-D echo findings associated w/ cardiac tamponade?
    • The 2-D echo findings are:
    • RV diastolic collapse
    • RA systolic collapse
    • R and L ventricle volume changes associated w/ respiration (these changes are better appreciated on M-mode studies)
  85. What are the Doppler findings associated w/ cardiac tamponade?
    Doppler evaluation of cardiac tamponade is aimed at measuring transvalvular flow velocities and detecting respiratory related changes in flow. Normally, mitral flow varoes less than 10%. In tamponade, however, the peak flow velocities may vary by as much as 40%. In general, tamponade may be indicated by respiration-related flow changes greater than 25% for mitral valve and greater than 50% for tricuspid valve.
  86. Of 2-d or Doppler findings which technique is more accurate?
    Doppler flow measurements correlate better w/ the clinical hemodynamics of tamponade than do 2-D findings, though 2-D findings are secondary findings to cardiac tamponade.
  87. Why might an echo-free space yield a false positive diagnosis of pericardial effusion?
    Loculated effusions are relatively rare. therefore anterior echo-free spaces usually represent excess pericardial fat, especially in obese patients.
  88. In what clinical settings might pericardial thickening be seen?
    Pericardial thickening may be seen in pts. who have chronic pericarditis or long-term history of steroid use. Such thickening may also be observed after radiation therapy.
  89. Is echocardiography the best noninvasive diagnostic technique for evaluating pericardial thickening?
    Echo is neither sensitive or specific in the diagnosis of pericardial thickening. This lack of reliability is due to the brightness and reflectivity of the pericardium. moreover, the pericardium lies at a depth of 15-20 cm, which is outside the optimal focal zone of most US systems. CAT scans or CMR is more accurate than echocardiography in determining the pericardial thickness.
  90. You have just completed an echo study on a patient who has all the echo and Doppler findings for tamponade. What is your next step?
    When you have a patient that you suspect is in tamponade you should find someone to do an immediate interpretation and get those results to the patients physician.
  91. What are the 3 most likely causes of fibrin or adhesions whithin the pericardial space?
    • The 3 most likely causes of fibrin or adhesions w/in the pericardial space are:
    • longstanding pericardial effusions
    • metastatic disease w/ pericardial involvement
    • hemorrhagic effusions that result in clot formation w/in the pericardial space.
  92. What is the most common etiology of constrictive pericarditis?
    The most common cause of constrictive pericarditis is, recurrent pericarditis, which may not have a clear underlying antecendent. Repeated cycles of inflammation, healing, and scarring cause the pericardium to become rigid and fibrotic.
  93. what are the primary physical findings associated with constrictive pericarditis?
    • The physical findings associated w/ constrictive pericarditis are:
    • dyspnea
    • ascites
    • a pericardial knock
    • jugular venous distention
  94. What is the cause of a pericardial knock?
    A pericardial knock occurs in early diastole and is caused by the abrupt cessation of ventricular filling?
  95. What is the definitive method for diagnosing constrictive pericarditis?
    Cardiac catheterization  is the definitive method for diagnosing constrictive pericarditis. It is better than echocardiography in this setting because catheterization documents the equalization of the R and L ventricular diastolic pressures.
  96. Describe the M-mode and 2-D echo findings associated w/ constrictive pericarditis?
    • In constrictive pericarditis, M-mode findings include flattened posterior wall motion during diastole, abnormal septal motion (early diastolic bounce), and 2 parallel lines w/in the pericardium.
    • 2-D findings include abnormal septal motion(early diastolic bounce), immobility of the  pericardium, and a dilated inferior vena cava
  97. Describe the Doppler findings associated w/ constrictive pericarditis?
    In constrictive pericarditis, Doppler findings include mitral and tricuspid regurgitation, a decreased early mitral inflow velocity (>25%) during inspiration, and a mitral inflow pattern similar to restrictive cardiomyopathy (large E-wave and a short A-wave).
  98. A 32 yr. old woman presents w/ atypical CP. Her BP is 112/50, height is 6', wt. 115 lbs. An echo is performed which shows a mitral valve prolapse. What is the most likely diagnosis for this pt.?
    Which add'l cardiac structures should be evaluated?
    • From the physical desc. of this pt. and the echo findings of MV prolapse she may have Marfan's Syndrome.
    • The AO valve and aorta should be evaluated for the presence of valvular regurgitation, aortic dilation and possible dissection
  99. A 39-yr. old male w/ no cardiac history enter's the Dr.'s office complaining of Flu-like symptoms for one week. A previously undocumented systolic murmur is heard. An echo reveals an abnl. thick anterior MV leaflet
    with mild mitral regurgitation. What is the most likely cause of these echo findings?
    What add'l. tests will be helpful in making the diagnosis in this pt.?
    • With a new murmur and the echo findings of mitral thickening in a young person, the most likely diagnosis is mitral valve endocarditis.
    • Blood cultures will be helpful in identifying the organism and a TEE will further assess the extent of the mitral leaflet thickening.
  100. A55-yr. old female complains of increasing dyspnea on exertion for 3 mos. Her physical examination reveals jugular distention, ascites, and a pericardial knock. What is the most likely clinical diagnosis?
    This pt. may well have constrictive pericarditis. The dyspnea, ascites, and jugular distention could all result from a restriction to diastolic filling. A pericardial knock is a classic physical finding associated w/ constriction.
  101. A32-yr. old female complains of fatigue and her chest x-ray reveals cardiomegaly. An echo is ordered, RA and RV enlargement are found. Add'l. there is a flattened intraventricular septum. M-mode findings of pulmonic valve mid-systolic closure and an absent A-wave. Identify the cardiac abnormality consistent w/ these findings?
    these echo findings are consistent w/ Pulmonary Hypertension. A microcavitation (saline contrast) study should be performed to rule out an atrial level shunt as the cause of this pulmonary hypertension.
  102. A 22-yr. old male complains of CP following exercise. An echo displays concentric LVH (2.5cm) and a small LV cavity size. Systolic motion of the mitral leaflet (SAM) and pericardial effusion are not observed. What is the most likely diagnosis for this pt.?
    What add'l noninvase diagnostic tests may help in diagnosing  this pt?
    • The findings of concentric LVH and a small LV cavity size is diagnostic for hypertrophic cardiomyopathy.
    • In order to identify the presence or absence of an obstructive component, an amyl nitrate challenge should be performed while the LVOT is interrogated by CW Doppler.
  103. A 38-yr. old male is sent to the echo lab for an evaluation after complaining of severe dyspnea on exertion for 2 months. M-mode findings include a dilated LV, increased EPSS, a B-notch on the mitral valve and overall hypocontractile left ventricular wall motion. What type of cardiac abnormality do these findings suggest?
    The M-mode findings of a dilated LV, increased EPSS, B-notch on the Mitral valves and overall hypocontractile LV wall motion identify a pt. with Dilated cardiomyopathy.
  104. After a chest x-ray revealed cardiomegaly, a 58-yr. old female, with a primary complaint of increasing dyspnea on exertion, is sent for and echo. The echo demonstrates LV hypertrophy with a bright myocardial appearance, LA enlargement and a small pericardial effusion. What is the pt.'s most likely cardiac diagnosis?
    LV hypertrophy (with a "bright," myocardium), LA enlargement and a small pericardial effusion are echo findings consistent w/ a diagnosis of infiltrative (restrictive) cardiomyopathy.
  105. A 56-yr. old female visits her doctor w/ the complaint of sob. Upon physical examination, a diastolic (rumbling) murmur is detected. The patient denies any history of rheumatic fever. What three cardiac abnormalities might be present in this patient?
    • 1.Mitral stenosis (or Tricuspid Stenosis). Even w/ no known history she may have had rheumatic fever as a child and now has rheumatic heart disease.
    • 2.Left atrial myxomas mimic mitral stenosis w/ regard to both physical findings and symptoms.
    • 3.Aortic regurgitation. If the aortic regurgitation jet hits the mitral valve anterior leaflet, the MV's opening can be restricted. As a result, a "rumbling" diastolic murmur (Austin Flint), rather than the typical "blowing" diastolic murmur, will be heard.
  106. A 31-yr. old male w/ a history of uncontrolled hypertension enters the emergency room w/ severe chest pain that he describes as "ripping". what is a likely cardiac diagnosis for the patient?
    Which noninvasive examination would be useful to confirm the diagnosis?
    • Although this patient may be experiencing a myocardial infarction, the history of hypertension and "ripping" chest pain also indicated the possibility of an aortic dissection.
    • Transesophageal echocardiography would be the next preferred noninvasive test as it is the fastest to perform the most sensitive method in diagnosing aortic dissection.
  107. An 18-yr. old male complains of "palpitations." His chest x-ray refeals cardiomegaly and as a result, an echo is ordered. The echo reveals the RA/RV enlargement. The tricuspid valve appears abnormally displaced towards the apex. What is the patients most probable cardiac abnormality? 
    What additional test should be performed in the echo lab?
    • This patient probably has Ebstein's Abnomaly. Often patients w/ Ebstein's Abnomaly are asymptomatic and this finding is a surprise when an echo is performed for something like evaluating a murmur.
    • A microcavitation (bubble or contrast) study should be performed to identify the presence or absence of an associated atrial septal defect.
  108. In the PSAX view, at the mitral valve and papillary muscle level how many segments is the LV divided into?
    Name those segments.
    In the PSAX level at the mitral valve and papillary level the LV is divided into 6 segments.

    • Those segments are:
    • Anteroseptal, Anterior wall, anterolateral wall, inferolateral wall, inferior wall and the inferoseptal.
  109. In the apical 4 chamber view, which two wall of the LV are seen?
    In the apical two chamber view, which two walls of the LV are seen?
    • In the AP 4 chamber wall you the LV wall segments seen are:
    • Anterolateral wall, and the inferoseptal septum.

    • In the AP 2 chamber view the LV walls seen are:
    • Anterior wall and the Inferior wall.
  110. Which walls of the LV are seen in the parasternal and the apical long axis view?

    Which two aortic valve leaflets are seen in these views?
    • In the parasternal and apical long axis views the walls of the LV that are seen are:
    • The anteroseptal wall and the inferolateral wall (AKA: posterior wall).
    • In the parasternal and apical long axis wall the cusps of the aorta that are seen are the:
    • The right and the non-coronary cusps.
  111. Where is the coronary sinus located in the parasternal long-axis view?

    Where is the coronary sinus located in relation to the descending aorta?

    How would you angle to view the coronary sinus in the apical 4-chamber view?
    • The coronary sinus lies in the posterior atrioventricular groove. This groove is located between the left atrial and left ventricular walls and lies posterior to the mitral valve. In the PLAX view, the coronary sinus can sometimes be seen as a small echo-free circle.
    • The coronary sinus is located anterior to the descending aorta. If the coronary sinus is dilated, it may be mistaken for the descending aorta.
    • From the apical 4-chamber view you would angle inferior in order to visualize the coronary sinus, which is located posterior to the mitral annulus.
  112. Why is it important to know the location of the coronary sinus and the descending aorta?

    What would cause the coronary sinus to be come dilated?
    • The coronary sinus and the descending aorta are important landmarks that can help differentiate pericardial effusions from pleural effusions. Pericardial effusions lie posterior to the coronary artery and anterior to the descending aorta. Pleural effusions lie posterior to the descending aorta.
    • A dilated coronary sinus could be the result of:
    • increased pressure in the RA (as in tricuspid regurgitation) or increased flow into the coronary sinus as in some congenital  malformations (as seen in Persistent Left Superior Vena Cava (PLSVC)).
  113. Name the three major coronary arteries?

    Where are the coronary arteries located on the surface of the heart?
    • The 3 main coronary arteries are:
    • RCA Right Coronary artery
    • LAD Left Anterior Descending artery
    • Cx Circumflex artery.
    • The last latter two arteries are branches of the Left main artery.

    • The coronary arteries are located on th eouter epicardial surface of the heart as follows:
    • RCA arises from the right aortic-root sinus, follows the R atrioventricular junction, and descends along the posterior inter ventricular groove.
    • The left anterior descending artery (LAD) follows the anterior interventricular groove. The circumflex (Cx) courses along the left atrioventricular junction.
  114. Name the cardiac walls supplied by each of the coronary arteries?
    • Normally the coronary arteries supply the following walls:
    • RCA-inferior wall, interoseptal, right ventricular apex, right ventricular free wall
    • LAD-anterior wall, anteroseptal, left ventricular apex
    • Cx-anterolateral wall, inferolateral wall
  115. While scanning a 43-yr. old man w/ a history of an old myocardial infarction, you notice that the anterior cardiac wall is akinetic. Which coronary artery is the most likely to have been involved in the infarction?
    The LAD  which supplies blood to the anterior cardiac wall, is most likely to have been involved. This artery also supplies the anterior portion of the ventricular septum and the left ventricular apex.
  116. In the apical 4 chamber view of a patient, the distal ventricular septum are the left ventricular apex are hypocontractile. Which coronary artery is most likely to be diseased?
    The LAD is the most likely choice. In some patient w/ distal septal hypocontractility, the proximal portion of the septum move normally because it is supplied by the right coronary artery.
  117. What are the normal systolic and diastolic pressures in the four cardiac chambers and great vessels?
    When is the Left Ventricular pressure at it's lowest?
    • Normal pressures are:
    • RA(mean)= 6mmHG
    • RV=25/5 mmHG
    • PA= 25/10 mmHg
    • LA(mean)= 10mmHg
    • LV= 120/7 mmHg
    • Aortic 120/80 mmHg
    • The LV pressure is lowest in early diastole just after the mitral valve opens. After that the LV pressure rises as the chamber fills in diastole.
  118. What is the normal mean pulmonary artery wedge pressure?

    How is the pulmonary artery wedge pressure determined?
    • The nl. mean pulmonary artery wedge pressure is 10 mmHg, which equals the left atrial pressure. The PA wedge pressure is NOT the same as the PA pressure.
    • A Swan-Ganz catheter is positioned in the pulmonary artery, and a small balloon is inflated at the catheters tip. The balloon is then floated and wedged into a smaller pulmonary artery. A pressure reading is obtained distal to the balloon. The inflated balloon prevents the tip of the catheter from sensing the pulmonary pressure, and the left arterial pressure is recorded as it is reflected across the pulmonary bed.
  119. To visualize the anterior wall of the left ventricle, which 2-D view would you use?

    To visualize the anterolateral wall of the left ventricle, which 2-Dview would you use?
    The anterior and inferior walls of the left ventricle are best visualized in the apical 2 chamber view.

    The anterolateral wall of the left ventricle is best visualized from the apical 4-chamber view. (The lateral wall can also be seen in the PSAX views, but the Apical 4 chamber view is best)
  120. On the electrocardiogram, at which point does the mitral valve normally close?

    On the electrocardiogram, at which point does the aortic valve normally open?
    The mitral valve normally closes approx. 60 milliseconds after the onset of the QRS complex, or about halfway though the QRS complex.

    The aortic valve normally opens at the end of the QRS complex. This answer takes in to account the delay between electrical  and mechanical systole, as well as the isovolumetric contraction time (between mitral closure and aortic opening).
  121. What is the relationship between electrical and mechanical systole?

    What is diastasis?
    • Mechanical systole follows electrical systole by approx. 12 milliseconds. this delay represents the time it takes for the electrical conductive impulse to spread and thereby cause myocardial contraction. The delay can best be evaluated during M-mode studies that examine the relationship between the electrocardiographic  pattern and valvular motion.
    • Diastasis denotes the middle portion of diastole, which occurs between early, rapid filling of the ventricles and the start of atrial contraction. The duration of diastasis varies w/ HR. Diastasis is quite long in patients w/ bradycardia and quite short in those w/ tachycardia.
  122. How much ventricular filling occurs during the passive phase of diastasis?

    Name the four phases of diastole?
    • At normal pressures, approximately 70% of ventricular filling occurs during the passive phase of diastole; atrial contraction accounts for the remaining 30% of ventricular filling. Of course, these percentages will change in patients with valvular abnormalities such as mitral stenosis or ventricular compliance problems such as hypertrophic cardiomyopathy.
    • The four phases of Diastole are:
    • isovolumetric relaxation time (closure of the AV to the opening of the mitral valve)
    • early rapid filling (passive)
    • diastasis
    • atrial contraction (active)
  123. What causes side lobe artifact?

    What is the best way to minimize side-lobe artifact?
    • Side-lobe artifacts are caused by strong reflectors outside the main ultrasound beam. these off-axis targets create reflections from the weaker extra ultrasound beams alongside the main beam.
    • The best way to minimize side-lobe artifacts is to decrease the overall gain, increase the reject level, or decrease the time gain (TGC's) in the area of strong reflectors (such as pericardium).
  124. What cardiac lesion is detected by injecting agitated saline material into the right side of the heart?

    Why does the saline contrast rarely appear in the left side of the heart?
    • Saline contrast material is injected to detect atrial shunts. It may also be used to document abnormal venous return and to detect right-sided intracardiac masses.
    • After being injected into a peripheral vein, saline contrast material advances into the right atrium. Alternatively, the contrast material may be injected directly into the right atrium through a Swanz-Ganz catheter. the mixed bubbles are too large to pass through the pulmonary bed. If the bubbles are seen in the LA and the LV within three-five heart beats after injection, an atrial level communication  should be suspected.
  125. How is the valsalva performed?

    How does the valsalva maneuver affect the heart?
    • The valsalva maneuver is performed in two phases: (strain and release)
    • inhaling half-way
    • closing mouth and nostrils
    • exhaling forcefully, straining against the closed mouth for about 5-10 seconds
    • opening mouth and exhaling
    • During the straining phase, the venous return decreases, so that the cardiac output diminishes and reflex tachycardia occurs. Once the strain is released, the venous return increases, along with right-sided cardiac pressures and cardiac output; a reflex bradycardia also occurs.
  126. What causes jugular venous distension?

    Name four cardiac problems, which commonly results in jugular venous distention?
    Jugular venous distension is caused by an increase in right atrial pressure.

    • Four cardiac problems, which commonly result in jugular distention are:
    • Cardiac tamponade
    • Pulmonary hypertension
    • Tricuspid stenosis
    • Constrictive pericarditis
  127. What is the best view to evaluate mitral stenosis w/ CW or PW Doppler?

    What is an important constant to remember for using the mitral  pressure half-time equation?
    • With CW or PW Doppler scanning, mitral stenosis is best evaluated from the cardiac apex. Because apical views allow the Doppler beam and the mitral stenotic  jet to be aligned in parallel fashion, these views yield accurate peak flow velocities.
    •  The most constant (empirical number) to remember for the mitral pressure half-time equation is 220. If the pressure half-time in milliseconds is greater than 220 then the MV area is less than one centimeter square (severe stenosis).
  128. How does amyl nitrate affect the circulation?


    How does inspiration affect venous return?
    • Amyl nitrate is a vasodilator that causes flushing, tachycardia, and hypotension. The cardiac output and ejection velocity are augmented. In general, murmurs associated with aortic or pulmonic stenosis are increased, while those associated with aortic  & mitral regurgitation are decreased.
    • Inspiration will cause an increase in venous return. Expiration will cause a decrease in venous return.
  129. When performing a 2-D echo exam, you notice that the image has very little gray-scale quality. Which controls could you adjust to increase the gray-scale quality?

    You also notice that the structures in the far field are very hard to see. Which controls could you adjust in order to see them better?
    • To increase the gray-scale quality, you would change the post-processing curve or the compress/reject control on most ultrasound machines. Also , you could check the monitor controls (brightness/contrast) and make sure that the transmit gain is not too high.
    • Increasing the TGC (Time Gain Compensation) control for the far field will brighten the structures in that area. Also switching to a lower frequency transducer will help with penetration and increase structures in the far field.
  130. In the Apical 4-chamber view, you have trouble differentiating an apical thrombus from an artifact. Name three things that can help you differentiate these two entities?
    • Three ways of differentiating an apical thrombus from an artifact are:
    • to change the depth settings, because range artifacts move with the change of depth.
    • to switch to a higher frequency transducer, preferably one with a short focal zone.
    • to decrease the transmit gain and time compensation gain controls in the near field to minimize the chest wall reverberations.
  131. In performing one-person CPR on an adult, what is the ratio of compression/breaths should you use?

    After you have determined that CPR is needed, what is the first step that you should take in performing this procedure?
    In one-person adult CPR the correct ratio of compressions/breaths is 15:2.

    Once you have determined the need for CPR the first thing you should do is establish an airway.
  132. How do you obtain an Apical 2-chamber view from an apical 4-chamber view?

    What is the easiest way to adjust the color Doppler gain?
    • From the Apical 4-chamber view, rotate the transducer counterclockwise approx. 30 degrees, until you see the LV (anterior and inferior walls), the mitral valve and the LA. If you see the aortic valve you have over rotated into the Apical long axis view.
    • With color flow Doppler turned on, increase the color gain until background Doppler noise appears on the display. Then decrease the color gain until the background noise disappears. Normal and abnormal flow should now be displayed in an optimal manner. If the Doppler display is still weak, switch to a lower frequency tx, decrease the depth of the field, or narrow the color sector to increase the frame rate.
  133. Describe the normal mitral valve anatomy?
    The mitral valve is a bileaflet valve situated between the LA and the LV. The valve's anterior leaflet is relatively long, lie's close to the aorta, and comprises one third of the valve's circumference. The posterior leaflet is shorter and is actually divided into three section's (scallops). Both the anterior and the posterior leaflets are attached to the ventricular papillary muscles by multiple chordae tendinae.
  134. Name the four classic M-mode findings associated with mitral stenosis?

    Describe the characteristic 2-D echo findings associated with mitral stenosis?
    • the classic M-mode findings associated with mitral stenosis are:
    • a decreased E-F slope
    • a decreased "E" wave amplitude
    • multiple reverberant echoes during diastole
    • anterior displacement of the posterior valve leaflet
    • the characteristic 2-D echo findings of mitral stenosis are:
    • left atrial enlargement
    • tethering of the tips of the mitral leaflets
    • thickening of the mitral leaflets, w/ decreased mobility
    • pulmonary hypertension and right atrial enlargement (which may be evident in severe mitral stenosis).
  135. What is the normal flow velocity (mean value and range) through the mitral valve, as documented by Doppler imaging in adults and children?

    Describe the changes seen in the Doppler trace in patients w/ mitral stenosis?
    • As documented by Doppler imaging, the normal flow velocity through the mitral valve in adults is a mean of 0.9 m/sec, with the range of 0.6-1.4 m/sec. In children, the flow velocity is slightly higher, having a mean of 1.0 m/sec and a range of 0.7-1.4m/sec.
    • The changes seen in the Doppler spectral trace in patients with mitral stenosis include:
    • an increase in flow velocity
    • an increase in flow turbulence (as detected by pulsed Doppler or color-flow studies)
    • a decrease in the rate of drop-off for the early diastolic slope (pressure half-time)
  136. How does mitral stenosis affect the left atrium?

    Why do patients with mitral stenosis usually develop atrial fibrillation?
    Mitral stenosis causes an increase in pressure in the LA, which results in LA enlargement.  Severe mitral stenosis can lead to pulmonary hypertension, RV failure, and RA enlargement.

    When the LA becomes dilated, the atrial muscle bundles are damaged, resulting in atrial fibrillation. Such fibrillation causes further atrial enlargement, as well as atrophy of the atrial muscle, and may lead to chronic atrial fibrillation.
  137. Name seven typical findings associated w/ mitral stenosis?

    What causes an opening snap?
    • Typical physical findings associated with mitral stenosis are:
    • A diastolic murmur (rumble)
    • an opening snap
    • atrial fibrillation
    • dyspnea on exertion
    • fatigue
    • orthopnea
    • heoptysys (spitting up of blood)
    • An opening mitral snap occurs shortly after the second heart sound (which signifies aortic and pulmonic closure). The snap is caused by the abrupt cessation of leaflet opening when the mitral valve is tethered.
  138. How does mitral stenosis affect the ventricles?

    How does it affect the great vessels?
    • Mitral stenosis has no effect on the left ventricle unless there is concurrent mitral regurgitation. Over time, mitral stenosis will cause elevation of the right sided cardiac pressures and will therefore lead to right ventricular enlargement, right atrial enlargement, and tricuspid regurgitation.
    • Mitral stenosis has not effect on the aorta. In the presence of severe, long standing mitral stenosis that results in PHTN, the pulmonary artery may become dilated.
  139. What is the most accurate way of calculating the mitral valve area?
    a. using M-mode echocardiography to determine the E-F slope
    b. performing 2-D planimetry  of the mitral orifice in the short axis view.
    c. determining the Doppler pressure half-time.
    • Performing 2-D planimetry  of the mitral valve orifice in the short-axis view is th emost accurate way  to measure the mitral orifice, provided that:
    • there is no echo dropout
    • the beam is perpendicular to the leaflets and is directed at the leaflet tips
    • the highest frequency transducer and the lowest gain settings possible are used
  140. A 2-Decho of a 55-yr. old woman with an opening mitral snap and a diastolic murmur reveals left atrial enlargement and thickening and tethering of the mitral valve leaflets. What is the most likely diagnosis?
    What other noninvasive test should be performed to further clarify the diagnosis?
    • The patient probably has rheumatic mitral stenosis. Although she has no history of rheumatic fever, she is young to have degenerative mitral disease. Moreover, her 2-D echo exhibits classic findings that indicate mitral stenosis.
    • To determine the severity of her mitral stenosis, this patient should undergo a Doppler examination, and the pressure half-time equation should be used to calculate her mitral valve area.
  141. What is the normal mitral valve area?


    What valve areas are associated with, mild, moderate, and severe mitral stenosis?
    • The normal mitral valve measures 4-5 cm2 in area and is therefore smaller than the tricuspid valve.
    • The valve areas associated with mild, moderate, and severe mitral stenosis are:
    • mild stenosis =1.5 to 2.5 cm2
    • moderate stenosis= 1 to 1.5 cm2
    • severe stenosis= < 1 cm2
  142. What are the main echo findings associated with chronic mitral regurgitation?

    What are the main echo findings associated with acute mitral regurgitation?
    • Mild, chronic MR may yield normal echo findings. Moderate-severe chronic MR usually causes LA enlargement, as well as LV dilitation and hypercontraclility, mitral deformities such as thickened leaflets, prolapse, or stenosis may also be present.
    • In pts. w/ acute MR, echo abnormalities such as valvular vegetations, torn chordae tendinae, and flail or partial flail mitral leaflets are often found. If the mitral regurgitation is ischemic in origin, the echo may show region wall-motion abnormalities when the patient is at rest.
  143. During pulsed Doppler examination, how does the MR appear on spectral trace?

    How is MR quantified w/ PW Doppler instrumentation?
    • During PW Doppler examination, MR appears as turbulent (broad spectrum) systolic flow w/in the LA. Because of the high velocity of the MR jet, the spectral trace will alias.
    • To quantify (semi quantify) MR w/ a PW Doppler instrument, the examiner "maps" the area of systolic turbulence in the LA. the larger the area of the regurgitant jet, the more severe the regurgeitation. It is important that the entire LA be mapped carefully, so that both the width and the length of the jet can be documented.
  144. How is MR quantified with Color Flow Doppler imaging?
    Color Flow Doppler imaging (which is similar to PW scanning) "maps" the area of the regurgitant flow. Unlike the PW Doppler approach, color-flow imaging show the regurgitant jet within a single cardiac cycle. The larger the size of the jet, the more severe the regurgitation. In determining the severity of the MR, the examiner must take into account the total size, length, and width of the jet.
  145. How does mitral regurgitation affect the atria?

    How does chronic chronic mitral regurgitation affect the ventricles?
    Mitral regurgitation can cause left atrial enlargement. The degree of such enlargement is usually proportional to the severity of the regurgitation. Trivial or mild regurgitation rarely results in atrial enlargement. Unlike mitral stenosis, in which atrial enlargement is due to increased pressure, mitral regurgitation causes atrial enlargement by producing volume overload.

    Mild mitral regurgitation has no noticeable effect on the ventricles. In contrast, moderate-severe chronic mitral regurgitation results in volume overload of the left ventricle. In absence of systolic dysfunction, the ventricle becomes dilated, and its wall becomes hypercontractile. Severe chronic mitral regurgitation can also cause increased pulmonary pressures and the right ventricular dilatation and hypertrophy.
  146. What is the definition of mitral valve prolapse, as documented by M-mode echocardiography?

    What is the definition of mitral valve prolapse, as documented by 2-D echocardiography?
    As documented by M-mode echocardiography, mitral valve prolapse is defined as posterior displacement of the mitral leaflets during systole. This displacement can either be holosystolic or mid-to-late systolic. The prolapsing leaflet should extend more than 2-3 mm below a line connecting the echocardiographic C-D points (beyond the annulus plane).

    As documented by 2-D echocardiography, mitral valve prolapse is defined as systolic displacement of one or both mitral leaflets into the left atrium in the parasternal or apical long-axis views.
  147. Why should you refrain from diagnosing mitral valve prolapse in the presence of a large pericardial effusion?

    Why should you refrain from diagnosing mitral valve prolapse from the apical four-chamber viewpoint?
    Diagnosing mitral valve prolapse in patients with a large pericardial effusion is more of a problem with M-mode than 2-D echocardiography. during late systole, when the entire heart moves in an anterior direction within the effusion, posterior movement of the mitral valve may be falsely interpreted as prolapse.

    Because the mitral-annulus is saddle-shaped, even normal mitral leaflets appear to prolapse into the left atrium when seen from the apical four-chamber view-point.
  148. Will rupture of a few mitral chordae tendineae cause any serious clinical problems?

    In what clinical setting is chordal rupture a potential source of diagnostic confusion?
    • Rupture of a few chordae tendineae rarely results in loss of leaflet support, so mitral regurgitation does not usually occur. Chordal rupture is typically seen in patients with  coronary artery disease or bacterial endocarditis.
    • In patients being evaluated for endocarditis, ruptured chordae tendineae may be difficult to distinguish from a vegetative mass. If available a previous echocardiogram is helpful for comparison.
  149. Will flail mitral leaflet cause significant hemodynamic problems? What symptoms will a patient with this condition probably present?Which of the two mitral papillary muscles has a higher incidence of rupture? Why?
    A flail mitral leaflet results in severe, acute mitral regurgitation. Because the left atrium does not have time to adapt to the increased hemodynamic volume, the left atrial pressure rises sharply, and patients often present with pulmonary edema. symptoms of acute pulmonary edema include sudden breathlessness; coughing up of pink, frothy liquid; and chest pain if the edema is caused by a myocardial infarction.

    The posteromedial papillary muscle has a higher rate of rupture than the anterolateral one. Whereas the posteromedial papillary muscle receives its blood supply from a single coronary artery (the RCA), the anterolateral papillary muscle receives a dual blood supply, from both the circumflex and the left anterior descending arteries.
  150. Patients with calcification or fibrosis of the mitral annulus commonly have mitral regurgitation. If the anatomy of the mitral leaflets is normal, what causes this regurgitation?
    The mitral regurgitation is probably caused by the fact that the annulus is now "fixed" and therefore unable to adapt to the left ventricular/atrial changes. Normally, the annulus is a flexible fibrous ring, whose shape changes to reflect alterations in the left ventricular geometry throughout the cardiac cycle.
  151. In mitral stenosis, a classic M-mode finding is flattening of the E-F slope. what causes this flattening?
    In M-mode imaging, the E-F slope of the  anterior leaflet represents  the rate of early diastolic filling of the left ventricle. Normally, the left atrium empties rapidly. In mitral stenosis, however , the filling time is prolonged, and this slow filling is reflected by the descent of the anterior leaflet. Attempts to quantify the degree of mitral stenosis on the basis of the E-F slope have not proved sensitive nor specific. The slope is affected by the severity of leaflet fibrosis, as well as left ventricle compliance, , the heart rate, and the motion of the mitral annulus during diastole.
  152. In patients with mitral stenosis, how does mitral regurgitation affect the peak flow velocity?

    Does mitral regurgitation affect the pressure half-time method of calculating the mitral valve area?
    • In patients with mitral stenosis, mild mitral regurgitation has no effect on the peak flow velocity. Moderate-severe regurgitation causes volume overload of the left ventricle and an increase in the mitral diastolic flow velocity.
    • Mild-moderate mitral regurgitation does not affect the pressure half-time method of calculating the mitral valve area. Whereas the peak mitral flow velocity may increase, the relationship between the peak and slope remain constant. Severe mitral regurgitation, with large increases in peak mitral flow velocity, may invalidate the pressure half-time method.
  153. Define systemic hypertension?

    What are the physical signs of systemic hypertension?
    Systemic hypertension is a persistent elevation of the systolic blood pressure to a greater than 140 mmHg and/or elevation of the diastolic blood pressure (to) greater than 90mmHg. these high-blood-pressure readings must be obtained on three separate occasions at least 1 week apart.

    Patients w/ uncomplicated hypertension are almost always asymptomatic. Headache, tinnitus, and dizziness may be seen in hypertensive patients but also occur in patients w/o hypertension.
  154. what are the vascular complications of systemic hypertension?

    What sometimes happens to the left ventricle late in the course of this disease?
    • The vascular complications of systemic hypertension include:
    • cerebrovascular accident (CVA)
    • kidney disease
    • coronary heart disease
    • congestive heart failure
    • peripheral vascular disease
    • aortic dissection

    Left ventricular failure sometimes occurs and the ventricle becomes dilated and hypocontractile just like any other dilated cardiomyopathy.
  155. What are the echocardiographic findings associated w/ systemic hypertension?

    What are the Doppler findings associated w/ systemic hypertension?
    • The echocardiographic findings associated with systemic hypertension include:
    • LVH
    • increased LV bulk
    • LA enlargement (usually mild)
    • possible aortic dilatation or dissection

    • The Doppler findings associated w/ systemic hypertension include:
    • an abnormal mitral inflow pattern featuring an A-wave greater than the E-wave and prolonged deceleration  that indicated decreased left ventricular relaxation
    • Aortic regurgitation (in the presence of Aortic dilatation)
  156. Define pulmonary hypertension?

    What are the physical signs of pulmonary hypertension?
    Pulmonary hypertension denotes a pulmonary artery pressure greater than 30 mmHg. It has many causes, including idiopathic, chronic mitral stenosis or regurgitation, pulmonary embolism, and Eisenmenger's syndrome.

    Physical signs of pulmonary hypertension include dyspnea during exertion (in early stages of the disease), dyspnea at rest ( in advanced stages), syncope, weakness, and precordial chest pain.
  157. What are the echocardiographic findings associated with pulmonary hypertension?

    What causes absence of the "a" wave?
    • In pulmonary hypertension, echocardiographic findings include:
    • an enlarged RA and RV
    • RVH
    • thickening of the interventricular septum
    • flattening of the interventricular septum in the short-axis view
    • an absent "a" wave and/or mid-systolic closure of the pulmonic valve (flying W) in M-mode.

    The"a" wave occurs when atrial contraction causes the RV pressure to increase, thereby deforming the closed pulmonic valve. In patients w/ pulmonary hypertension, the pulmonary artery pressure is so high that, even during atrial contraction, the pulmonic leaflets do not move. Therefore, no "a" wave is produced.
  158. When the LV short-axis view demonstrates flattening of the interventricular septum how can you determine if the problem is volume or pressure related?
    Flattening of the interventricular septum will be seen in both volume and pressure overloaded RV. In fact, there is never a purely volume or pressure overloaded situation-there is always some combination of the two. One way to determine which is dominant, volume or pressure is to watch the motion through the cardiac cycle. Most of the time the septum will return to normal in systole (rounded up) when volume overload is predominate. the septum will stay flattened, most of the time, when the problem is due to pressure.
  159. How is the pulmonary artery pressure calculated on the basis of the tricuspid regurgitant jet?

    How is the pulmonary artery pressure assessed using the Doppler pulmonary artery acceleration time?
    Right ventricular systolic pressure (RSVP) can be calculated by adding the tricuspid regurgitation velocity (converted to mmHg by 4v2) and estimated right atrial pressure (by dimension and collapse of the IVC).

    The pulmonary artery pressure can be calculated from the pulmonary artery Doppler spectral trace by measuring the systolic acceleration time. the normal systolic acceleration time is greater than 120 msec, as measured from the onset of flow to peak velocity. In patients w/ pulmonary hypertension, the acceleration time is decreased. In general, an acceleration time less than 75 msec indicates at least moderate pulmonary hypertension (in adults).
  160. Describe the three main classifications of cardiomyopathy?

    What is the typical echocardiography appearance of each classification of cardiomyopathy?
    • The three main classifications of cardiomyopathies are:
    • hypertrophic (with/without obstruction)
    • dilated (congestive)
    • restrictive (infiltrative)
    • In hypertrophic cardiomyopathy, the ventricular walls are thickened (symmetrical or asymmetrically) and the ventricular chambers are reduced in size.
    • In dilated cardiomyopathy, the ventricles are enlarged and contractility is decreased. One or both chambers may be affected.
    • In restrictive cardiomyopathy, the ventricles are hypertrophied, the cardiac chambers are nearly normal in size, and the myocardium appears bright.
  161. What is the most common cause of hypertrophic cardiomyopathy?

    What causes systolic anterior motion (SAM) of the mitral valve?
    • The most common cause of hypertrophic cardiomyopathy is genetic. Just over half of all patients with such cardiomyopathy have an autosomal dominant gene trait. In other patients, the disease appears to occur spontaneously.
    • Systolic anterior motion (SAM) of the mitral valve is caused by the venture effect. The left ventricular outflow tract is narrowed by septal hypertrophy. As ejection occurs, the velocity in the narrowed outflow tract increases, creating a lower pressure region above the mitral, drawing the mitral leaflets toward the septum.
  162. What are the physical findings associated with obstructive cardiomyopathy?

    List the common abbreviations associated with hypertrophic cardiomyopathies and the state what each abbreviation means?
    • The physical findings associated with obstructive cardiomyopathy are:
    • dyspnea on exertion; this is the most common symptom, occurring in 90% of asymptomatic patients.
    • angina (which occurs in 75% of asymptomatic patients)
    • syncope
    • sudden death
    • ASH-asymmetrical septal hypertrophy
    • HCM-hypertrophic cardiomyopathy
    • HOCM- hypertrophic obstructive cardiomyopathy
    • IHSS-idiopathic hypertrophic subaortic stenosis (AKA-HOCM)
  163. How does the administration of amyl nitrate affect the murmur associated with hypertrophic obstructive cardiomyopathy?

    What happens to such a murmur when the Valsalva maneuver is performed?
    The murmur associated with hypertrophic obstructive cardiomyopathy is usually harsh, systolic, and of a crescendo-decrescendo type. It is best heard between the cardiac apex and the left sternal border. In response to amyl nitrate, this murmur will increase.

    Like amyl nitrate, the Valsalva maneuver causes an increase in the systolic murmur associated with hypertrophic obstructive cardiomyopathy.
  164. Describe the M-mode appearance of hypertrophic obstructive cardiomyopathy.
    • In M-mode, hypertrophic obstructive cardiomyopathy is usually characterized by:
    • left ventricular hypertrophy (symmetric or asymmetric)
    • a small left ventricle cavity
    • systolic anterior motion of the mitral valve
  165. In the evaluation of hypertrophic cardiomyopathy, how does 2-D echocardiography examination differ from M-mode examination?
    2-D echocardiographic examination shows the extent of the hypertrophy better than M-mode imaging (especially when the hypertrophy is asymmetric) and also reveals apical hypertrophy. In addition, 2-D examination shows the hypertrophied muscle's "ground glass" appearance, which results from disarray of the myocardial fibers. Other findings that are better seen with 2-D imaging include calcification/fibrosis of the mitral annuls mitral valve thickening, and septal scarring where the leaflet strikes the interventricular septum.
  166. How is PW Doppler examination useful in assessing hypertrophic cardiomyopathy?

    How is CW Doppler useful in the assessment of hypertrophic cardiomyopathy?
    PW Doppler examination, performed from the apical window, localizes the site obstruction. When the sample volume is moved from the cardiac apex toward the aortic valve and the obstruction is encountered, the flow velocity increases and changes from laminar flow to turbulent on the spectral Doppler trace.

    CW Doppler records the peak gradient both at rest and after provocation with the Valslva maneuver or amyl nitrate. The spectral trace often shows late-peaking systolic jet typical of obstructive cardiomyopathy.
  167. How is color-flow Doppler examination helpful in the assessment of hypertrophic cardiomyopathy?
    The color-flow Doppler technique (similar to the PW technique)can indicate an area of obstruction in the left ventricular outflow tract by turning the color display into a mosaic pattern where turbulence is detected. CF Doppler is also helpful in detecting and quantifying mitral regurgitation, which is common in hypertrophic obstructive cardiomyopathy.
  168. Name five common causes of dilated cardiomyopathy?

    What is the most common cause of dilated cardiomyopathy in the United States?
    • Causes of dilated cardiomyopathy include:
    • idiopathy
    • infection (viral, bacterial, fungal, or parasitic)
    • toxicity resulting from alcohol abuse, lead poisoning, AIDS, or drugs such as Adriamycin
    • a peripartum state

    According to some studies, alcohol is the most common cause of dilated cardiomyopathy in the United States. Alcoholic cardiomyopathy most commonly occurs in 30-55 year old men who have a longer than 10 year history of heavy drinking.
  169. What is Chagas' disease?

    What are the echocardiographic findings in Chagas' disease?
    Chagas' disease results in cardiomyopathy. It is casued by a parasite bite (Reduvid Bug) that leads to a myocarditis years later. chagas' disease is epidemic in south America, affecting 14 million people. A history of living or traveling in South America would be an important finding in a patient with unexplained cardiomyopathy.

    Like most dilated cardiomyopathies, regardless of the cause, Chagas' disease results in a dilated and hypocontractile left ventricle. In some patients the septum has normal contractility and it's the apical and posterior walls that are affected.
  170. Name 6 classic M-mode findings associated with dilated cardiomyopathy?
    • Classic m-mode findings associated with dilated cardiomyopathy include:
    • a dilated left ventrcile
    • increased E-point septal separation (EPSS)
    • hypocontractile left ventricular wall motion
    • a B-notch on the Mitral valve
    • a double-diamond mitral valve (when the valve closes in mid-diastole)
    • decreased aortic root motion
  171. In dilated cardiomyopathy, what advantages does 2 -D echocardiography have over M-mode imaging?

    How does color flow Doppler imaging contribute to the diagnosis of dilated cardiomyopathy?
    In dilated cardiomyopathy, 2-D echocardiography allows better global assessment of ventricular function and the atrial size and better detection of thrombi than does M-mode imaging. 2-D approach is also useful for excluding valvular heart disease, and it may detect pericardial effusions, which are common in these patients.
  172. In patients with dilated cardiomyopathy, why is biventricular enlargement often present?

    Why is atrial enlargement present in these patients?
    Biventricular enlargement is often seen in patients who have dilated cardiomyopathy from myocarditis. The infectious organism damages both the right and left ventricular myocardium.

    Whatever the etiology, the diastolic pressure increases as the ventricle enlarges. To encounter this increased pressure, the left atrium also enlarges.
  173. Describe the prominent feature of post-cardiac transplant patient's echocardiogram.
    The prominent echocardiographic feature in a post-cardiac transplant is enlargement of both the right and left atria. The attaching donors ventricles and atria to the recipients atria causes these double atria. This way the recipient's atrial connections (IVC,SVC, and Pulmonary veins) are not disturbed.
  174. Name four causes of restrictive cardiomyopathy?

    Which cause is the most common one?
    • Causes of restrictive cardiomyopathy include:
    • amyloidosis
    • sarcoidosis
    • hemochromatosis
    • glycogen storage disorders

    Amyloidosis is the most common cause of restrictive cardiomyopathy.
  175. Why is restrictive cardiomyopathy sometimes referred to as infiltrative cardiomyopathy?

    How does restrictive cardiomyopathy affect the atria? Why does it have this effect?
    The disease is restrictive with respect to physiology(i.e. the hypertrophied ventricular wall restricts filling) but the disease process is infiltrative, because it involves amyloid or sarcoid deposits w/in the myocardium.

    In restrictive cardiomyopathy, the atria enlarge to a greater extent than in any other type of cardiomyopathy. This biatrial enlargement is caused by elevated filling pressures of both ventricles.
  176. Why does the myocardium appear so bright in patients with restrictive cardiomyopathy?
    The high reflectivity (ground-glass appearance) of the myocardium results from the amyloid deposits (in amyloidosis) or iron deposits ( in hemochromatosis). Although this "ground-glass" appearance is classic for infiltrative cardiomyopathies and should be included on any exam question with today's improved transducers and equipment many easy to image patients will have very bright myocardium.
  177. Describe the appearance of restrictive cardiomyopathy, as documents by 2-D echocardiographic images?
    • In restrictive cardiomyopathy, 2-D echocardiographic imaging usually reveals:
    • left ventricular hypertrophy
    • a bright, reflective myocardium
    • a ventricular cavity that is normal or near normal in size
    • atrial enlargement
    • a small-moderate pericardial effusion (cardiac tamponade is rare)
  178. What are the Doppler findings associated with restrictive cardiomyopathy?
    In restrictive cardiomyopathy, Doppler findings include mitral and/or tricuspid regurgitation (which is usually mild) and abnormal inflow patterns. The classic Doppler pattern consists of a tall E wave, with rapid deceleration, and a very small A wave.
  179. Name the five types of cardiac wall motion that can be seen echocardiographically?
    • The five types of wall motion are:
    • normal
    • hypercontractile (exaggerated) motion
    • hypocontractile (sluggish) motion
    • akinesia (absence of motion)
    • dyskinesia (motion opposite to the normal pattern)
  180. Describe the differences between a true ventricular aneurysm and a pseudoaneurysm.

    Which type of aneurysm has a higher rate of rupture?
    • The walls of a true ventricular aneurysm include all three cardiac layers-the endocardium, myocardium, and epicardium. although the lesion may contain thrombus, the aneurysm's mouth is wider than it's body. In contrast, a pseudoanuerysm  is a ruptured portion of the ventricle. Its wall includes the epicardial and pericardial layers. these aneurysms almost always contain thrombus.
    • Ventricular pseudoaneurysms rupture more frequently than true aneurysms and are considered surgical emergencies.
  181. What 8 potential complications of a myocardial infarction?

    Which structural complication is most commonly seen?
    • Complications of myocardial infarction include:
    • arrythmias
    • aneurysm formation
    • pseudoanuerysm formation
    • pericardial effusion
    • mural thrombus
    • papillary muscle dysfunction
    • ventricular septal defects
    • death

    Aneurysm formation is the most common complication of myocardial infarction, occurring in 8-15% of cases.
  182. Name six risk factors for coronary artery disease?
    • Six risk factors of coronary artery disease are:
    • Smoking
    • male gender
    • positive Family history
    • increased age
    • hypertension
    • hyperlipidemia
  183. What is Dressler's syndrome?
    Dressler's syndrome is a form of pericarditis that is also known as the postmyocardial infarction syndrome. It occurs 6-8 weeks after an infarction and is characterized by positional chest pain and a pericardial effusion.
  184. Describe the post infarction changes that the myocardium might exhibit, as documented by 2-D imaging.
    • 2-D echocardiographic imaging may show the following post infarction changes in the myocardium:
    • fixed segmental wall-motion abnormalities
    • myocardial scarring, as indicated by increased brightness and decreased wall thickening
    • ventricular aneurysms, which are seen in up to 22% of cases
    • a pericardial effusion, which is usually small
    • left ventricular mural thrombi
  185. What information from an echocardiogram does a surgeon need before operating to reduce a left ventricular aneurysm?
    • A surgeon would like to know:
    • size of aneurysm
    • location
    • presence of thrombus in the aneurysm
    • movement of other walls

    of these the movement of the other walls is most important. If there is not enough salvageable myocardium then the patient is not a surgical candidate.
  186. 2 weeks after sustaining a inferior wall myocardial infarction, a 44 yr. old man returns to the ER w/ SOB, CP, and a new systolic murmur. What is the most likely diagnosis? Why?
    The most likely diagnosis is rupture of a papillary muscle, producing severe acute mitral regurgitation. 90% of papillary ruptures occur after infarction of the inferior left ventricular wall.

    The postero-medial muscle has one coronary artery feeding it while the antero-lateral has dual blood supply (LAD and Circ)
  187. What is the basic rule regarding thrombus in the ventricles?
    A basic rule is that thrombus does not form on a ventricular wall that moves normally. Approximately 40% of acute myocardial infarctions will produce a left ventricular thrombus. Most thrombi of this type form between the sixth and tenth days after infarction and are found in the left ventricular apex.
  188. How does a right ventricular infarction appear on a 2-D echocardiographic image?
    In autopsy series, a right ventricular infarctions account for up to 70% of inferior myocardial infarctions but are clinically recognized in only 3% of these cases. Right ventricular free-wall akinesis is the most sensitive echocardiographic indicator of a right ventricular infarction. Other, less specific findings include right ventricular dilatation, tricuspid regurgitation, and paradoxic septal motion.
  189. In 2-D echocardiography, how do the findings produced by ischemic cardiomyopathy differ from those caused by viral cardiomyopathy?

    What is the role of Doppler examination in ischemic heart disease?
    A dilated, hypocontractile left ventricle may appear in both cases. In ischemic cardiomyopathy, however, the right ventricle is often unaffected. In viral cardiomyopathy, the right ventricle is dilated and hypocontractile.

    • Doppler examination is important in assessing the complications of myocardial infarction, particularly ischemic mitral regurgitation, flail mitral valve with severe regurgitation, and ventricular septal defect.
    • Right ventricular infarctions my be accompanied by tricuspid regurgitation, although this sign is not specific for such infarctions.
  190. What advantage does exercise echocardiography have over regular exercise electrocardiography in detecting coronary artery disease?
    In detecting single-vessel coronary artery disease, regular exercise electrocardiography has a sensitivity of about 60%. In comparison exercise echocardiography has a sensitivity of about 90%. In patients w/ multi-vessel coronary disease, this method's sensitivity is even higher.
  191. Name five cardiac problems that a penetrating wounds to the heart might cause?
    • Five cardiac problems that penetrating wounds to the heart might cause:
    • Myocardial rupture
    • Traumatic ASD or VSD
    • Valvular disruption
    • Pericardial effusion
    • Coronary artery trauma
  192. In cases of penetrating wounds to the heart what is the problem with the presence or absence of pericardial effusions?
    The problem with the presence or absence of pericardial effusions is that only 50-70% of patients with cardiac trauma will have an effusion. Patients w/ gunshot wounds were the ones with pericardial effusions 50% of the time. So, absence of pericardial effusions doesn't mean the heart is not involved (especially in gunshot wounds).
  193. What is a myxoma?

    What are the most likely locations for cardiac myxomas?

    What percentage of cardiac myxomas recur?
    A myxoma is a benign tumor composed of mucopolysaccharide (mucous) cells. Most myxomas have a soft, gelatinous appearance. Some are multilobulated, and others are more encapsulated.

    In 75% of cases, cardiac myxomas form in the left atrium. The right atrium is the next most common location, followed by the left and right ventricles. Multiple myxomas occur in approximately 5% of patients.

    5-14% of cardiac myxomas recur after resection. Recurrence is usually at the original tumor site but may also involve other intracardiac sites.
  194. What symptoms and physical findings are associated with a left atrial myxoma?

    Which other cardiac abnormality does the symptoms and physical findings of a left atrial myxoma mimic?
    In left atrial myxoma, symptoms are related to obstruction of the mitral orifice. They include dyspnea, fatigue, weakness, and a diastolic murmur.

    The symptoms and physical findings of a left atrial myxoma mimic those of a patient with rheumatic mitral stenosis?
  195. What is the most common primary malignant cardiac tumor?

    What is the most common benign cardiac tumor seen in children?
    The most common primary malignant cardiac tumor is the sarcoma. The tumors account for about 25% of all primary cardiac tumors and are second only to myxomas in frequency. Sarcomas have a varying histologic features and may take the form of an angiosarcoma (the most common histologic type), a rhabdomyosarcoma, or a fibrosarcoma.

    Rhabdomyomas are the most common benign cardiac tumor seen in children. These tumors, which are derived from cardiac muscle cells, are frequently multiple, are intracavitary and/or intramyocardial, and may obstruct cardiac inflow or outflow. Fibromas are the second most common cardiac tumor seen in children and are often intramyocardial.
  196. Name five technical maneuvers that can differentiate a cardiac mass from an artifact during echocardiography?
    • To differentiate a cardiac mass from an artifact, the examiner can:
    • decrease the overall transmit gain and time gain compensation controls
    • use multiple cardiac windows (all masses should be documented in two or more echocardiographic views)
    • change the depth of the view, thereby possibly changing the position of range artifacts
    • switch to a higher-frequency transducer to improve resolution
    • inject contrast materials, which may help identify right-sided masses by outlining the lesion.
  197. If you see a mobile mass in the inferior vena cava extending into the right atrium what should you be suspicious that this patient might have?
    masses in the inferior vena cava could be a thrombus but you should be suspicious for renal cell carcinoma until that has been ruled out with further tests.
  198. How does atrial fibrillation affect the mitral valve, as documented by M-mode imaging?

    How does atrial fibrillation affect the left ventricle, as seen by 2-D imaging?
    • As documented by M-mode imaging; atrial fibrillation affects the mitral valve by causing;
    • irregularity of the length of the diastolic filling periods
    • absence of an A wave
    • fibrillatory waves on the mitral leaflet during diastoles
    • (these waves are best seen on the anterior leaflet)

    The effect of atrial fibrillation on the left ventricle is less obvious on 2-D echocardiography than on M-mode imaging but may include variations in ventricular size as the diastolic filling period changes.  Atrial fibrillation may also cause changes in the amplitude of the ventricular contraction, with longer diastolic intervals having more forceful ejections.
  199. Describe the appearance of Wolff-Parkinson-White syndrome (WPW), as documented by M-mode imaging.
    In M-mode imaging, the appearance of WPW syndrome varies, depending on whether the syndrome is type A (involving preexcitation of the posterior left ventricular wall) or type B (involving preexcitation of the anterior Right ventricular wall). Type A is characterized by early systolic contraction (notching) of the posterior left ventricular wall. In contrast, type B is characterized by abnormal septal motion, such as posterior motion (notching) during early systole.
  200. How do frequent premature ventricular contractions (PVC's) affect the aortic Doppler spectral trace?

    How does atrial flutter affect the mitral valve, as documented by M-mode imaging?
    The Doppler spectral trace shows the premature ventricular contractions effect aortic flow. When a premature beat occurs, the diastolic interval is shortened, and little or no flow is seen. Because a compensatory pause ensues, involving a longer than normal diastole, the next beat will have a higher velocity as a result of the enlarged ventricular volume.

    In M-mode imaging, the effect of atrial flutter on the mitral valve is characterized by rapid, regular flutter waves or undulations, which correspond to the atrial contractions.
  201. In assessing wall-and valve-motion irregularities caused by arrhythmias  and conduction disturbances, which is preferred: M-mode imaging or 2-D echocardiography?
    Why?

    What are the typical M-mode findings associated with bundle branch block?
    M-mode imaging is the best means for assessing arrhythmic changes and conduction disturbances. Compared to 2-D echocardiography, M-mode imaging has a much higher sampling rate (temporal resolution), so it can record subtle or rapid changes in wall or valve motion.

    Typical M-mode findings associated with bundle branch block are limited to abnormal ventricular septal contraction patterns. Left bundle branch blocks often produce early systolic posterior deflection of the interventricular septum. Right bundle branch block is associated with abnormal septal depolarization (from left to right), and no abnormal septal motion is present.
  202. What is the most common congenital cardiac anomaly?

    What is one of the most prevalent cyanotic congenital cardiac lesions?
    Ventricular septal defect (20% to 30% of all defects) is the most common congenital anomaly. Bicuspid aortic valves are more common in the general population (1-2%), but are considered a variant of normal unless stenotic.

    Tetralogy of Fallot is one of the most prevalent cyanotic congenital cardiac lesions.
  203. What are the 3 possible deformities involved in an atrioventricular septal defect?

    What type of patients are a high risk for having an atrioventricular septal defect?
    Atrioventricular septal defects can involve:

    • a primum atrial septal defect
    • a ventricular septal defect
    • a cleft mitral valve

    Patients with Downs Syndrome (Trisomy 21) are at high risk for atrioventricular septal defects.
  204. What congenital cardiac lesion is commonly associated with pulmonic stenosis?

    What congenital cardiac lesion is associated with bicuspid aortic valve?
    Pulmonic stenosis is commonly associated with a ventricular septal defect.

    Bicuspid aortic valve is commonly associated with aortic coarctation.
  205. Name and describe the four main types of atrial septal defects?
    • The four main types of atrial septal defects are:
    • Secundum defects, which are located in the mid-septal area and are the most commonly observed type (incidence, 70%).
    • Primum defects, which are located in the inferior septum, close to the atrioventricular valves, and are the second most common type (incidence, 20%).
    • Sinus venosus defects, which are located near the entrance of the superior vena cava. these defects are also associated with anomalous pulmonary venous return for approximately 8% of atrial septal defects.
    • Coronary septal defects, which are located in the inferior septum, close to the coronary sinus, and account for only 2% of atrial septal defects.
  206. What is the main direction of flow across an atrial septal defect?

    In color Doppler imaging, which echocardiographic view is best for detecting flow across an atrial septal defect?
    At normal right and left ventricular pressures, the direction of flow across an atrial septal defect is left-to-right.

    In color-flow Doppler imaging, the best echocardiographic view for detecting left-to-right flow across an atrial septal defect is the sub-costal four chamber view. In this view, shunt flow is parallel to the ultrasound beam, so echocardiographic sensitivity is optimal.
  207. If the primary flow direction through a shunt is a left-to-right, why are contrast echocardiographic studies so sensitive in detecting atrial shunts?

    What effect does an atrial septal defect have on  the cardiac chambers?
    • Even when shunt flow is primarily left-to-right, a small right-to-left component is usually present at end-diastole. If the contrast study yields a negative result, performing another injection during the release phase of the Valsalva maneuver may produce a positive result, because the right-sided pressure will undergo a transient increase.
    • Shunt flow across an atrial septal defect causes the right ventricular diastolic volume and the pulmonary blood flow to increase.In addition to the right ventricular dilatation resulting from volume overload, paradoxic wall motion or septal flattening is seen in the parasternal short-axis view. The ventricle is rarely affected.
  208. What is a persistent left superior vena cava?
    A persistent left superior vena cava is a congenital venous malformation where the left arm drains into the coronary sinus instead of the superior vena cava. Returning blood from the left arm goes to the right atrium, it just takes a detour through the coronary sinus.
  209. What is the prominent 2-D finding in a patient with a persistent left superior vena cava?

    How is the diagnosis of a persistent left superior vena cava confirmed?
    The prominent 2-D finding in patients with persistent superior vena cava is a dilated coronary sinus, usually best seen in the parasternal long-axis view.

    The diagnosis of a persistent left superior vena cava is confirmed by performing a saline contrast study with the injection into the patients left arm. This way bubbles (contrast) will appear in the dilated coronary sinus first and then the right atrium and ventricle.
  210. Describe the most common types of ventricular septal defects?
    Ventricular septal defects take the form of:

    • a perimembranous defect near the aortic valve (this is the most common type, accounting for 75% of ventricular septal defects)
    • an inlet defect in the posterior portion of the septum, close to the tricuspid valve.
    • an outlet or supracristal defect in the right ventricular outflow tract.
    • a muscular defect low in the ventricular septum is completely surrounded by muscular tissue. Multiple muscular ventricular septal defects may coexist.
  211. Which technique is more useful in detecting a ventricular septal defect: Doppler or contrast echocardiography?
    Doppler imaging is more useful than contrast echocardiography in detecting a ventricular septal defect. A pulsed or color-flow approach should be used to locate the defect, and continuous-wave studies are preferred for estimating the gradient between the ventricles. Because blood is almost always shunted from left to right, saline contrast echocardiography has limited success in detecting these defects.
  212. What is the primary direction of flow across a ventricular septal defect?

    Describe the physiology of Eisenmenger's syndrome?
    At normal ventricular pressures, the primary direction of flow across a ventricular septal defect is left-to-right.

    In Eisenmenger's syndrome, a long-standing left-to-right shunt, in the form of a ventricular or atrial septal defect, causes pulmonary hypertension. When the pulmonary pressures exceed the systemic (LV) pressures the shunt reverses direction, becoming right-to-left.
  213. How does a ventricular septal defect affect the cardiac chambers?
    Small ventricular septal defects do not usually affect either the right or left cardiac chambers. Moderate-to-large ventricular septal defects cause the left ventricle to enlarge as a result of the pressure/volume overload that flows through the pulmonary system. The left atria may dilate as well, the RV and /RA are rarely affected, this is due to the flow be shunted almost directly into the PA.
  214. What is a patent ductus arteriosus (PDA)?

    Which echocardiographic view and technique is best for assessing a patent ductus arteriosus?
    • Patent ductus arteriosus (PDA) is an anomaly in which the ductus arteriosus fails to close after birth. The ductus arteriosus is a fetal communication between the pulmonary artery and the descending aorta. In the fetal circulation, this ductus shunts blood away from the lungs, toward the aorta.
    • The para-sternal chort axis view, at the level of the aorta, is the best echocardiographic view for detecting the patent ductus arteriosus. Color flow Doppler imaging is the best technique for detecting shunt flow. Usually, blood flows through the shunt in a left-to-right direction (from the aorta to the pulmonary artery) during diastole.
  215. In coarctation of the aorta, which portion of the aorta is usually involved?

    How is the severity of an aortic coarctation quantitated?
    Although coarctation may involve any portion of the aorta, this anomaly usually occurs just distal to the origin of the left subclavian artery. This area is known as the aortic isthmus.

    The severity of obstruction at the site of the coarctation can be quantified with continuous wave Doppler. From the aortic suprasternal notch, the Doppler beam can be aligned with the descending aorta, allowing the peak gradient to be measured. Forward flow may also be present across the coarctation during diastole (such flow is known as diastolic runoff).
  216. What is Ebstein's anomaly?

    What congenital lesion is often associated with Ebstein's anomaly?
    Ebstein's anomaly is a congenital malformation and apical displacement of one or more tricuspid leaflets. Although the degree of the leaflet displacement varies, at least part of the morphologic right ventricle becomes "atrilaized". The tricuspid annulus is positioned normally, and moderate-to-severe tricuspid regurgitation is usually present.

    Secundum atrial septal defects are often associated with Ebstein's anomaly.
  217. Which echocardiographic views are the most helpful in assessing Ebstein's anomaly?

    What conduction disturbance is associated with Ebstein's anomaly?
    Either the apical or the subcostal four-chamber view is best for visualizing displacement of the tricuspid leaflets in Ebstein's anomaly. When the anomaly is mild, these views also allow accurate measurements of the distance between the insertions of the mitral and tricuspid leaflets. If these insertions are separated by more than 8mm, some degree of tricuspid-leaflet malposition is present.

    Patients with Ebstein's anomaly have a increased incidence of Wolff-Parkinson-White (WPW) syndrome, usually with the right accessory pathway.
  218. Describe the three types of aortic dissection.

    Which types are usually associated with aortic regurgitation?
    • The three types of aortic dissection are:
    • Type 1, which originates in the proximal ascending aorta and extends into the descending aorta (Stanford A, Debaky 2)
    • Type 2, which originates in the ascending aorta and is confined to the ascending aorta (Stanford A, Debaky 1)
    • Typer 3, which originates in the descending aorta and extends into the abdominal aorta. Alternatively, in rare cases, type 3 dissection extends towards the aortic arch (Debaky 3).

    Aortic regurgitation is seen in both type 1 and type 2 aortic dissection, because these types involve the aortic root
  219. Describe the 2-D echocardiographic appearance of a sinus of Valsalva aneurysm.
    • Sinus of Valsava aneurysms involve the right coronary sinus in 70% of cases, the noncoronary sinus in 25% of cases, and left coronary sinus in 5% of cases.
    • 2-D echocardiography best reveals such aneurysms in the parasternal short axis-view, at the aortic level. Because aneurysms rarely involve more than one sinus, identification of the affected sinus is facilitated by comparing the size and wall thickness of all three sinuses. An aneurysmal sinus is larger than normal and has thin walls. These aneurysms can rupture, and if this complication oocurs, color flow Doppler imaging is very useful in identifying the direction and volume of flow.
  220. What are the echocardiographic findings associated with aortic dissection?

    What is the best echocardiographic technique for evaluating aortic dissection?
    Echocardiographically,aortic dissection is associated with local or generalized dilatation of the aorta. The aortic wall has a double-layered appearance (either anteriorly or posteriorly). The intimal flap appears as a thin, mobile structure that divides the aorta into a true and a false channel.

    Transesophageal echocardiography is the best technique for diagnosing an aortic dissection and evaluating its extent.
  221. What echocardiographic finding may help differentiate miscellaneous aortic dilitaiton from Marfan's syndrome?
    Most patients (70%) with Marfan's syndrome have mitral valve prolapse. Marfan's syndrome is a connective-tissue disease in which the chief cardiac abnormalities are aortic aneurysms, aortic regurgitation, and mitral valve prolapse. Aortic dissection is common and accounts for the majority of premature deaths.
  222. In the presence of a ventricular septal defect, how would you calculate the right ventricualr systolic pressure (RVSP)?
    • In the absence of aortic stenosis, the right ventricular systolic pressure (RVSP) can be calculated by 1) measuring the gradient between the ventricles on the basis of the flow velocity across the ventricular septal defect (VSD); 2) converting the result to mmHg with the Bernoulli equation (4v2); and 3) subtracting this gradient from the systolic blood pressure (SBP).
    • The equation is RVSP=SBP-VSD gradient.
  223. What is the color flow technique?
    The colr flow Doppler technique is a pulsed method for recording and displaying flow information, as well as anatomic images (m-mode or 2-D). Color flow displays usually involve 2-D images; by convention, flow toward the probe appears red, and flow away from the probe appears blue. The display indicated the flow velocity and direction by changing the hues of these two colors.
  224. In color-flow imaging, what is a pulse packet?

    Why are pulse packets important?
    To accurately determine the direction and velocity of blood flow, a group of the ultrasound pulses is transmitted, received, and compared with respect to phase shift. This group of pulses is called a pulse packet.

    The larger the pulse packet, the more accurate the flow-related information. The bad news is, large packets size takes longer to transmit and receive so image frame-rate decreases.

    Note: For satisfactory blood-flow sensitivity, it is generally best to use a large packet size and to gain back an acceptable frame rate by decreasing the depth or narrowing the color sector.
  225. What is the continuous wave Doppler mode?

    What are the main advantages and disadvantages of the continuous wave Doppler mode, compared to the pulsed-wave Doppler mode?
    In the continuous wave Doppler mode, two transducer elements (or sets of elements) are used to record flow. One element continuously transmits ultrasound signals, and the other element continuously receives such signals. The received signals are analyzed for Doppler frequency shifts, and the resulting information is displayed on the screen.

    The main advantage of the continuous wave Doppler mode is its ability to record high-velocity flow without aliasing. The disadvantage of this mode is that, although flow along the ultrasound beam is recorded the location of this flow is not known.
  226. What is the most important part of the Doppler equation?
    The most important part of the Doppler equation is the cosine of the angle theta. Theta denotes the angle between the ultrasound beam and the moving target (blood). As long as the angle is less than 20 degrees, the calculated flow velocity is accurate. When the angle is greater than 20 degrees, the velocity is underestimated by the Doppler equation.
Author
jbryers69
ID
284606
Card Set
ARDMS registry
Description
ARDMS Registry
Updated